All about flooble | fun stuff | Get a free chatterbox | Free JavaScript | Avatars    
perplexus dot info

Home > Numbers
True for nine and up (Posted on 2017-03-24) Difficulty: 3 of 5
If a set N9 = {1, 2, 3, 4, 5, 6, 7, 8, 9} of 9 numbers is split into two subsets, then at least one of them contains three terms in arithmetic progression.
The statement is not true for a set N8 of 8 integers.

Seems obvious?

Prove it.

No Solution Yet Submitted by Ady TZIDON    
No Rating

Comments: ( Back to comment list | You must be logged in to post comments.)
Solution By hand | Comment 2 of 3 |
The N9 partition must have a subset with at least 5 numbers.  If I didn't make any mistakes, there are four possibilities where this subset has no APs.  In each case the other subset has an AP in its 4 numbers instead.

12489 / 3567
12679 / 3458
12689 / 3457
13489 / 2567

The N8 partition is easy.  Here are the ways I found
1256 / 3478
1368 / 2457
1458 / 2367

Edited on March 25, 2017, 9:30 am
  Posted by Jer on 2017-03-24 12:39:01

Please log in:
Login:
Password:
Remember me:
Sign up! | Forgot password


Search:
Search body:
Forums (1)
Newest Problems
Random Problem
FAQ | About This Site
Site Statistics
New Comments (17)
Unsolved Problems
Top Rated Problems
This month's top
Most Commented On

Chatterbox:
Copyright © 2002 - 2024 by Animus Pactum Consulting. All rights reserved. Privacy Information